Sie sind auf Seite 1von 25

MEDICAL

safeguards to guarantee the safety of the patient


and impose liability to the practitioner who
JURISPRUDENCE through this act or omission causes damage or
injury to the health and welfare of the patient.
The right to regulate the practice of medicine is
Deals with the aspect of law and legal concepts based on the police power of the state.
in relations with the practice of medicine
INCLUDES
o Licensure and regulatory laws;
o Physician-patient-hospital relationship
LICENSURE AND REGULATORY LAWS
together with the other paramedical
personnel, their rights, duties and
Administrative Bodies
obligations;
BOARD OF MEDICAL EDUCATION
o Liabilities for non-compliance with the
o Primarily concerned with the
law
standardization and regulation of
medical education
PURPOSES
PROFESSIOAL REGULATIONS COMMISSIONS
To protect the public from charlatans;
o To have general supervision and
To promote professionalism and foster
regulation of all professions requiring
professional interrelationship;
examinations which included the
To develop awareness of the rights, duties and
practice of medicine
obligations of the patient, physician, and the
BOARD OF MEDICINE
hospital;
o Its primary duties are to give
To control the increasing number of medical
examinations for the registration of
malpractice suits against physicians;
physicians and supervision, control and
To explain the purpose and procedure of certain
regulation of the practice of medicine
legislation;
To study the need to amend, repeal our health BOARD OF MEDICIAL EDUCATION
care laws in harmony with the recent scientific Composition
and social development
Chairman Secretary of Education
Members Secretary of Health
ADVERSARIAL TRIAL SYSTEM
Director, Bureau of Private
Philippine courts is a court litigation where there
Schools
is competition of inconsistent version of facts
Chairman, Board of Medicine
and theories in law during trial;
Representative, PMA
Each party to the contest is given equal
Council of Deans, APMC
opportunity to investigate the case, gather and Dean, UP-College of Medicine
present all proofs in support of his allegation, Functions
and give argument that his contention is correct; 1. To determine and prescribe the requirements of
Ultimate purpose is for a just solution. admission into a recognized college of Medicine;
it often undermines the pursuit of truth as the 2. To determine and prescribe requirements for
opposing parties seek to win at all cost without the minimum physical facilities;
the obligation to reveal the facts which may be 3. To determine and prescribe the minimum
detrimental to their case. The lawyer aims to win number and qualifications of teaching
the fight not to help the court discover facts or personnel;
establish the truth. 4. To determine and prescribe the minimum
required curriculum;
SOURCES OF LAW 5. To authorize the implementation of
Constitution experimental curriculum;
Laws enacted by the legislative body 6. To accept applications for admission to a medical
Decrees, Orders, Proclamation, Letters, CA, BP, school;
RA 7. To select, determine and approve hospitals for
Administrative acts, orders, Rules and training;
Regulation 8. To promulgate, prescribe and enforce the
Local customs necessary rules and regulations
Generally accepted principles of International
law PROFESSIONAL REGULATIONS COMMISSION
Composition
LAW AND PRACTICE OF MEDICINE Commissioner
The state must maintain high standard of Two Associate Commissioner
practice of setting up rules and regulations with Exercise of Power and Functions of the Commission
regards to qualifications and procedure for the Exercise general administrative, executive and
admission to the profession. These are legal policy-making functions for the whole agency
o Documentary evidence confirmed
BOARD OF MEDICINE by the DFA showing that his
Composition countrys existing laws permit
Six members appointed by the president from a citizens of the Philippines to practice
list submitted by the Executive Council of the medicine under the same rules and
PMA regulations governing citizens
Qualifications thereof (RECIPROCITY RULES)
Natural-born citizen 4. Holder of certificate of registration
Duly-registered physician - No issuance to any candidate who has been:
In the practice of medicine for at least 10 years o Convicted by a court of competent
Of good moral character and of recognized jurisdiction of any crime involving
standing in the medical profession as certified by moral turpitude;
PMA o Found guilty of immoral or
Not a member of nay faculty of any medical dishonourable conduct after
school (including any pecuniary interest) investigation by the Board of
Powers, Functions and Responsibilities Medicine;
1. To determine and prepare the contents of the o Declared to be of unsound mind
licensure examinations;
2. To promulgate such rules and regulations for the SCOPE OF EXAMINATION
proper conduct of the examinations, correction Preliminary
and registration; Anatomy and Histology
3. To administer oath; Physiology
4. To study the conditions affecting the practice of Biochemistry
medicine; Microbiology and Parasitology
5. To investigate violations, issue summons, Final
subpoena and subpoena duces tecum; Pharmacology and Therapeutics
6. To conduct hearings or investigations of Pathology
administrative cases filed before them; Medicine
7. To promulgate decisions on such administrative Obstetrics and Gynecology
cases subject to the review of the Commission; Pediatrics and Nutrition
8. To issue certificate of registration; Surgery and Ophthalmology,
9. To suspend, revoke or reissue certificate of Otorhinolaryngology
registration for causes provided by law or by the Preventive Medicine and Public Health
rules and regulations promulgated; Legal Medicine, Ethics and Medical
10. To promulgate, with the approval of PRC, rules Jurisprudence
and regulations in harmony with the provisions
of the Medical Act of 1959 and necessary for the
proper practice of medicine
PRACTICE OF MEDICINE
ADMISSION TO THE PRACTICE OF MEDICINE
Prerequisites What is the practice of medicine?
1. Minimum age requirement o It is a privilege or franchise granted by
- At least 21 years of age the State to any person to perform
2. Proper Educational Background medical acts upon compliance with the
- Requirements for Admission in the College law, that is, the Medical Act of 1959 as
of Medicine amended which has been promulgated
o Holder of a Bachelors degree; by the State in the exercise of police
o Not convicted of any crime involving power to protect its citizenry from
turpitude; unqualified practitioners of medicine.
o Certificate of Eligibility from the o It is diagnosing and applying and the
Board of Medical Education; usage of medicine and drugs of curing,
o Good moral character mitigating, or relieving bodily disease or
3. Examination Requirements conditions.
- Must have passed the corresponding Board
Examination ACTS CONSTITUTING THE PRACTICE OF MEDICINE
- Preliminary Examination (pursuant to Sec. 10, Art. III of the Medical Act of 1959 as
o At least 19 years of age amended):
o Of good moral character A. Who shall for compensation, fee, reward in any
o Have completed the first two years form paid to him directly or through another, or
of the medical course; even without the same, physically examine any
- Final or Complete Examination person, and diagnose, treat, operate or prescribe
o Citizen of the Philippines or of any any remedy for human disease, injury,
country who has submitted deformity, physical, mental, psychical condition
competent and conclusive
or any ailment, real or imaginary, regardless of than 5 years, or by both such fine and
the nature of the remedy or treatment imprisonment, in the discretion of the court
administered, prescribed or recommended;
B. Who shall by means of signs, cards, QUALIFIED TO PRACTICE MEDICINE IN THE PHILIPPINES
advertisement, written or printed matter, or 1. Those who have complied with the prerequisites
through the radio, television or any other means to the practice of medicine in accordance with
of communication, either offer or undertake by Sec. 8, Art. III, Medical Act of 1959 as amended
any means or method to diagnose, treat, 2. Those who can have limited practice without any
operate, or prescribe any remedy for human certificate of registration in accordance with Sec.
disease, injury, deformity, physical, mental or 12, Art. II, Medical Art of 1959 as amended:
psychical condition; - Exclusive consultation in specific and
C. Who shall falsely use the title of M.D. after his definite cases;
name, shall be considered as engaged in the - Attached to international bodies to perform
practice of medicine certain definite work in the Phils;
- Commissioned medical officers stationed in
EXEMPTIONS the Phils in their own territorial jurisdiction;
By DECISIONS OF COURTS are not considered to - Exchange professors in special branches of
constitute practice of medicine: medicine;
a. One who take BP reading - Medical students who have completed the
b. Application of medicated massage first four years of medical course, graduates
c. Hospital of medicine and registered nurses who may
d. Nurse anesthesist be given limited and special authorization by
By PROVISIONS OF LAW are not considered to the DOH;
constitute practice of medicine (Sec. 11, Art. III, 3. Balikbayan Physician pursuant to PD 541,
Medical Act of 1959 as amended): allowing former Filipino professionals to practice
i. Any medical students duty enrolled in an their respective profession in the Philippines
approved medical college - Proviso
ii. Dentist o Of good standing prior to their
iii. Physiotherapist departure and in their adopted
iv. Optometrist country;
v. Any person who renders any service o Have registered with PRC and paid
gratuitously in cases of emergency or in their professional fee
places where the services of a physician, o Pay the corresponding income tax
nurse or midwife are not available 4. Foreign physicians qualified to practice by
vi. Any person who administers or recommends Reciprocity Rule or by endorsement;
any household remedy as per classification 5. Medical Students pursuant to Sec. 11(a) and Sec.
of existing Pharmacy Laws 12(d), Art. III, Medical Act of 1959 as amended;
vii. Clinical psychologist with the prescription 6. Limited practitioner of medicine
and direct supervision of a physician - Those that are governed by specific licensure
viii. Prosthetist laws

FAITH HEALING RATIONALE WHY ARTIFICIAL PERSONS CANNOT


There is nothing in the Medical Act of 1959 PRACTICE MEDICINE
exempting it from the definition of the acts Cannot be subjected to licensure examinations
which constitute practice of medicine as required by law
Related to constitutional guarantee to religious Practice of medicine may be employed and
freedom (freedom to believe and freedom to act controlled by unqualified physicians
in accordance with ones belief) Professional relationship between the patients
Acted in pursuance of his religious belief and and the physician will be impaired
with the tenets of his church he professes, not Deprivation of free choice of physicians
deemed to be a practice of medicine but part of
his religious freedom PHYSICIAN
is a person who after completing his
ILLEGAL PRACTICE OF MEDICINE secondary education follows a prescribed course
Practice of medicine by any person not qualified of medicine at a recognized university or medical
and not duly-admitted to perform medical acts school, at the successful completion of which, is
in compliance with law legally licensed to practice medicine by the
Penalties responsible authorities and is capable of
Pursuant to Sec. 28, Art. IV, Medical Act of 1959 undertaking the prevention, diagnosis, and
as amended Any person found guilty of illegal treatment of human illness by the exercising
practice shall be punished by a fine of not less independent judgment and without
than P1,000 or more than P10,000 with supervision. (WHO)
subsidiary imprisonment in case of insolvency or
by imprisonment of not less than 1 year, no more
According to Justice Malholm SOME INSTANCES WHERE THERE IS NO PHYSICIAN-
CONSTITUTION is that written instrument PATIENT RELATIONSHIP BY DECISIONS OF COURTS
enacted by direct actions of the people by which 1. Pre-employment PE for purposes of determining
the fundamental powers of the government are whether an applicant is suitable for
established, defined, and by which these powers employment;
are distributed among the several departments 2. PE for eligibility for insurance;
for their safe and useful exercise for the benefit 3. Physician appointed by court to examine the
of the body politic. accused;
4. In performing an autopsy;
Purpose 5. Casual consultation in an unordinary place.
To prescribe the permanent framework of a
system of government, to assign to the several PSYCHOLOGICAL PATTERNS OF PHYSICIAN-PATIENT
departments their respective powers and duties, RELATIONSHIP
and to establish certain first principles on which Activity-Passive Relation
the government is founded. (11Am. Jur. 606) o No interaction between physician and
patient because the patient is unable to
ESSENTIAL PARTS OF A CONSTITUTION contribute activity. This is characteristic
Constitution of Liberty in an emergency cases when the patient
Constitution of Government is unconscious.
Constitution of Sovereignty Guidance-Cooperation Relation
o Patient is conscious and suffering from
AMENDMENT isolated or piecemeal change in the pain, anxiety and other distressing
constitution while REVISION is the revamp or the symptoms, he seeks help and willing to
rewriting of the entire instrument cooperate. The physician is in a position
of trust.
Mutual Participation Relation
o It is in the nature of a negotiated
PHYSICIAN-PATIENT RELATIONSHIP agreement between equal parties.

CONTRACT is the meeting of minds between DUTIES AND OBLIGATINS IMPOSED ON THE PHYSICIAN IN
two persons whereby one binds himself with THE PHYSICIAN-PATIENT RELATIONSHIP
respect to the other, to give something or to
render some service (Art. 1305, NCC) 1. He should possess the knowledge and skill of
which an average physician is concerned
NATURE OF THE RELATIONSHIP - General practitioner vs Specialist
Consensual based on mutual consent both 2. He should use such knowledge and skill with
parties ordinary care and diligence
Fiduciary based on mutual trust and - locality rule the standard of cares is
confidence measured by the degree of care in the
locality
REQUISITES OF A CONTRACTUAL RELATIONSHIP - similar locality rule diligence is
Consent manifested by the meeting of the determined when the other physicians in the
offer and the acceptance upon the thing and the locality or similar locality could have acted
cause which are to constitute the contract (Art. the same way
1319NCC) - national standard of care the diligence is
Object the subject matter of the contract which determined on what is applicable on a
is the medical service which the patient wants to national standard basis
be rendered to him by his physician 3. He is obliged to exercise the best judgment
4. He has the duty to observe utmost good faith.
Cause the consideration or the factor that
instigated the physician to render the medical
N.B.
service to the patient, which could be
remuneratory or an act of liberality Physician-Patient relationship does not imply
guaranty or any promise that the treatment will
FORMS OF PHYSICIAN-PATIENT RELATIONSHIP be successful.
1. Expressed explicitly stated orally or in writing It does not imply any promise or guaranty that
2. Implied the existence can be inferred from the the treatment will benefit the patient
acts of the contracting parties. Inferred by law as It does not imply any promise or guaranty that
a matter of reason and justice for their acts or the treatment will produce certain result
conduct It does not promise or guaranty that the
treatment will not harm the patient
It does not promise that the physician will not
commit errors in an honest way
DUTIES AND OBLIGATIONS IMPOSED ON THE PATIENT
ON THE COURSE OF THE PHYSICIAN-PATIENT RIGHT TO CHOOSE PATIENTS
RELATIONSHIP Any person who is giver right to practice
1. He must give an honest medical history medicine is not obliged to practice medicine
2. He must inform the physician of what occurred Cannot be compelled to accept professional
in the course of treatment employment
3. He must cooperate and follow the instructions, NB: The law does not give any qualification the
orders and suggestions of the physician right of the physician to choose his patient,
4. He must state whether he understands the however, the CODE OF MEDICAL ETHICS and RA
contemplated course of action 6615 provides otherwise in cases of emergency.
5. He must exercise the prudence to be expected of Related provisions
an ordinary patient under the same o Art. II, Sec. 2, Code of Ethics
circumstance xxxfree to choose whom he
will servexxxalways respond
STAGES OF PHYSICIAN-PATIENT RELATIONSHIP to any request for is assistance
Commencement in emergencyxxx
It is the very time the physician is obliged to o Art. II, Sec. 3, Code of Ethics
comply with the legal duties and obligations to In cases of emergency, xxxa
his patients. physician should administer at
Termination least first aid treatment and
It is the time when the duties and obligations by then refer to a more qualified
a physician to his patient ceases. The following and competent physicianxxx
are some ways of termination of the o Sec. 1, RA 6615
relationship: All government and private
1. Recovery of the patient or when the hospitalsxxx are required to
physician considers that his medical services render immediate emergency
will no longer be beneficial to the patient medical assistancexxx
2. Withdrawal of the physician provided: o Sec. 24, No. 12, Medical Act of 1959
a. With consent of the patient xxxAlthough the ethical rule
b. Patient is given ample time and obliges a physician to attend to
notice an emergency, his failure to
3. Discharge of the physician by the patient respond to it may not make him
4. Death of either party liable if in so doing, there is a risk
5. Incapacity of the physician to his life.
6. Fulfillment of the obligations stipulated in Refusal of a physician to attend
the contract to a patient in danger of death is
7. In emergency cases, when the physician of not a sufficient ground for
choice of the patient is already available or revocation or suspension of his
when the condition of emergency ceases registration if there is a risk to
8. Expiration of the period as stipulated the physicians life
9. Mutual agreement for its termination
RIGHT TO LIMIT HIS MEDICAL PRACTICE
Field of specialty
Private clinic or hospital
RIGHTS OF PHYSICIAN Within a political/geographical boundary
Certain days of the week/hours of the day
Inherent rights Certain class of people
To choose patients With due regard to dictate of conscience
To limit practice of medicine Retirement
To determine appropriate management Imposed by the public, religion, professional
procedures ethics, medical society, law, contract
To avail of hospital services
Incidental rights RIGHT TO AVAIL OF HOSPITAL SERVICES
Right of way while responding to emergency Right to determine the appropriate
Right of exemption from execution of management procedure
instruments and library o Doctrine of Superior Knowledge
To hold certain public/private offices The physician has superior
To perform certain services knowledge and the patient just
To compensations follows orders or instructions
To membership in medical societies and usually places himself in the
Rights generally enjoyed by every citizen command and control of the
Pursuant to the provisions of Art. III, bill of rights, physician
Philippine Constitution 1987
Right of way while responding to the call of o This kind of fee is unethical because the
emergency amount wagers with the unforceable
contingencies
RIGHT OF EXEMPTION FROM EXECUTION OF
INSTRUMENTS AND LIBRARY METHOD OF COLLECTION OF PAYMENT FOR MEDICAL
Rule 39, Sec. 12, Rules of Court SERVICES
Right to hold certain public and private offices 1. Extra judicial billing or referral to a bill
which can only be filled up by physicians collection agency
Right to perform certain services 2. Judicial methods
Right to membership in medical societies Facts to be proven in Court
o Any qualified medical practitioner has a. Physician employed is duly qualified and licensed
the right to become a member of the b. The physician has rendered professional service
PMA through one of its component to the patients
society c. The professional fee demanded is reasonable
o Membership in a medical society may be d. The person liable for the payment is the
voluntary and involuntary defendant.

Philippine Medical Care Act of 1969 (RA 6111 as The obligation to pay devolves on the patient
amended) provides that membership to the PMA is a himself provided he is of legal age, of sound mind
requirement before a physician can practice medicine and has the capacity to enter into a contractual
under the Medicare. relation.
If the patient dies or becomes legally
RIGHT TO COMPENSATION incapacitated to pay, medical fee shall be made
Based on the physician-patient contractual from the following persons in order:
relationship 1. Spouse
Existence of friendship does not imply gratuitous 2. Descendants, of the nearest degree
services 3. Ascendants, of the nearest degree
4. Brothers and sisters
Art. IV, Sec. 2, Code of Ethics NB.
should willingly render gratuitous service to a o Implied promise to pay the physician by
colleague, to his wife and minor children or even the benefactor of the medical services
parents provided the latter are aged and being rendered in emergency cases
supported by the colleague. He should however,
be furnished the necessary traveling INSTANCES WHERE THE PHYSICIAN CANNOT RECOVER
expensesxxxthis provision shall not apply to PROFESSIONAL FEES:
physicians who are no longer in the active 1. Agreement that the service is gratuitous
practicexxx 2. In government charity hospitals, health centers
Doctrine of Unjust Enrichment and other similar health units
o No one must enrich himself at the 3. Rendered in private charitable institutions if
expense of others expressly gratuitous to the indigent patients
o service rendered service paid 4. Waiver on the part of the physician
5. Breach of contract
KINDS OF MEDICAL FEES 6. When the physician cannot charge the patient
1. Simple Contractual Fee specifically stating the pursuant to the Code of Ethics
value of such medical service, either orally or in 7. Those covered by Phil Health
writing 8. Medical services rendered under a contract of
2. Retainer Fee measured by the space of time employment unless expressly provided
rendered by patient otherwise
3. Contingent Fee depends upon the failure or the
treatment instituted
4. Dichotomous Fee (Fee splitting) the physician
may require the services of a person who may RIGHTS OF PATIENTS
act as agent to solicit patients, and the agent will
share in the medical fee 1. Right to give consent to diagnostic and
o Art. III, Sec. 5, Code of Ethics treatment procedures
xxxsolicitation of patient, 2. Right to religious belief
directly or indirectly, through 3. Right of privacy
solicitors or agents, is unethical 4. Right to disclosure of information
5. Straight Fee for the amount tendered by the 5. Right to confidential information
patient to the physician, the latter shall be 6. Right to choose his physician
responsible for the payment of hospital bill, lab 7. Right of treatment
fees, medicines and other incidental expenses; 8. Right to refuse necessary treatments
RIGHT TO GIVE CONSENT TO DIAGNOSTIC PROCEDURES 5. Other person who may give consent having
Obligations of the Physician to inform the patient: substitute parental authority
1. Diagnosis Subject matter is legal
2. General nature of the contemplated procedure The subject matter or procedure applies to the
a. Risk involved patient and which the patient consented must
b. Prospect of success not be that which the law penalizes or against
c. Potential danger if not applied public policy
d. Alternative methods of treatment NB:
patient is the final arbiter of what must be Consent of minor is not valid if the procedure will
done with his body. not benefit him
Bases of Consent Expressed refusal of a minor to surgery shall not
1. The physician-patient relationship is fiduciary in prevail over the existing emergency
nature. Doctrine of parents patria, the court may grant
2. Patients right to self-determination. consent for the minor
3. Contractual relationship.
Purposes RIGHT TO RELIGIOUS BELIEF
1. To protect the patient from Art. III, Sec. 5, Philippine Constitution
unnecessary/unwarranted procedure applied to No law shall be made respecting an
him without knowledge establishment of religion or prohibiting the free
2. To protect the physician from any consequences exercise thereof. The free exercise and
for failure to comply with legal requirements enjoyment of religious profession and worship,
Instances When Consent is Not Necessary without discrimination or preference, shall
1. In cases of emergency, there is an implied forever be allowed. No religion test shall be
consent or the physician is privilege because required for the exercise of civil or political
he is reasonable entitled to assume consent rights.
2. When the law made it compulsory for everyone
to submit to the procedure. RIGHT OF PRIVACY
Requisites of a Valid Consent RIGHT OF DISCLOSURE OF INFORMATION
1. Informed or enlightened consent The physician-patient relationship being
2. Voluntary fiduciary in nature, the physician is obliged to
3. Subject matter must be legal make full and frank disclosure to the patient or
Forms of Consent any person who may act on his behalf all the
1. Expressed consent written or oral pertinent facts relative to his illness
2. Implied consent may be deduced from the Art. 1339, Civil Code states that xxxfailure to
conduct of the patient disclose pacts, when there is duty to reveal
Scope of the Consent them, as when the parties are bound by
1. General or Blanket consent confidential elations, constitutes fraud.
2. Limited or conditional consent RIGHT OF CONFIDENTIAL INFORMATION
3. Non-liability or exculpatory clause Statutory Privileged Communication
Informed/Enlightened Consent o Pursuant to the Rules of Court, Rule 130,
Awareness and assent Sec. 24, a person authorized to
Full disclosure of facts and willingness of the practice medicine, surgery or obstetrics
patient to submit cannot in a civil case, without the
Quantum of Information Necessary to Form the Basis of consent of the patient, be examined as
a Valid Consent to any information which he may have
1. Nature of his condition acquired in attending such patient in a
2. Nature of proposed treatment or procedure professional capacity, which information
3. Possible alternative methods was necessary to enable him to act in
4. Risk involved that capacity, and which would blacken
5. Chances of success or failure the character of the patient.
Ethical/Professional Confidential Information
Consent must be given freely or voluntarily o Pursuant to Art. II, Sec. 6, Code of
Medical Ethics, the medical practitioner
Persons who can give consent should guard as a sacred trust anything
1. Patient that is confidential or private in nature
2. If patient is minor, consent must be obtained that he may discover or that may be
from the parents communicated to him in his professional
3. In the absence of parents, consent of the relation with his patients, even after
grandparents must be obtained, paternal death. He should never divulge this
grandparents having preference confidential information, or anything
4. In the absence of parents and grandparents, that may reflect upon the moral
eldest brother or sister, provided one is of age character of the person involved, except
and not disqualified by law to give consent
when it is required in the interest of Penalty: reprimand, suspension, to revocation of
justice, public health or public safety. license
Some instance where confidentiality is not Criminal
applicable: An act of omission which constitute a crime by
1. When such disclosure is necessary to serve the physician
the best interest of justice. Laws: Revised Penal Code and other special laws
2. When the disclosure will serve public health Penalty: imprisonment and/or fine
3. When the patient waives its confidentiality Civil
Awarded against a physician to compensate for
RIGHT TO CHOOSE HIS PHYSICIANS the injury he suffered on account of the
RIGHT TO TREATMENT physicians act or omission as a breach of the
In emergency cases, the patient has the right to contractual relationship of both parties
treatment Laws: Civil Code of the Philippines and other
Sec. 1, 1st par, RA 6615 related laws
Provides that xxxall government and private Art. 100, RPC states the Every person criminally
hospital or clinics duly licensed to operate are liable is civilly liable. Penalty: damages
required to render immediate medical
assistance and to provide facilities and medicine ADMINISTRATIVE LIABILITIES
within its capabilities to patients in emergency Quantum of evidence needed: substantial
cases who are in danger of dying and/or suffered evidence, such relevant evidence as a
serious physical injuriesxxx reasonable mind might accept as adequate to
Art. II, Sec. 3, Code of Medical Ethics support a conclusion
In cases of emergency, wherein immediate Administrative Due Process:
action is necessary, a physician should 1. Right to hearing
administer at least 1st aid treatment and refer 2. Tribunal must consider the evidence presented
the patient to a more qualified and competent 3. Decision must have something to support itself
physician if the case does not fall within is 4. Evidence must be substantial
particular line. 5. Decision must be based on the evidence
RIGHT TO REFUSE TREATMENT adduced at the hearing, or at least contained in
In the legal sense, every man of adult age and of the record and disclosed to the parties
sound mind has the right to determine what 6. The Board or its judges must act on its or their
must be done in his own body. A man is the independent consideration of the facts and the
master of his own self and may expressly law of the case, and not simply accept the views
prohibit a life-saving surgery or medical of a subordinate in arriving at a decision (Ang
treatment. Tibay vs CIR)
Doctrine of parens patria, the State has the right
to assume guardianship when the child is GROUND FOR ADMINISTRATIVE LIABILITIES
neglected by the parents to have the child Sec. 24, Art. III, Medical Act of 1959 as amended
treated, and parents have no right to base it on Personal Disqualifications
religious beliefs or any other grounds 1. Immoral or dishonorable conduct
When the law provides for treatment, the 2. Insanity
patient has no right to refuse treatment. 3. Gross negligence, ignorance or incompetence
The social commitment of the physician is to resulting in an injury to or death of the patient
sustain life and relieve suffering. Where the 4. Addiction to alcoholic beverages or to any habit-
performance of ones duty conflict with the forming drug rendering him incompetent to
other, the choice of the patient, or his family or practice medicine
legal representative if incompetent to act on his Criminal Acts
own behalf, should prevail. In the absence of the 1. Conviction by a court of competent jurisdiction
patients choice or authorized proxy, the of any criminal offense involving moral turpitude
physician must act in the best interest of the 2. Fraud in the acquisition of the certificate of
patient. registration
3. Performance of or aiding in any criminal abortion
4. Knowingly issuing false medical certificate
5. Aiding or acting as dummy of an unqualified or
LIABILITIES OF PHYSICIAN unregistered person to practice medicine
Unprofessional Conduct
Administrative 1. False or extravagant or unethical advertisement
Right to practice is temporarily withdrawn from wherein other things than his name, profession,
the physician limitation of practice, clinic hours, office and
A valid exercise of the police power of the State home address, are mentioned
Laws: Medical Act of 1959 as amended including 2. Issuing any statement or spreading any news or
the Code of Ethics and Rules and Regulations of rumor which is derogatory to the character and
the PRC
reputation of another physician without
justification
3. Violation of any of the Code of Ethics as CIVIL LIABILITIES
approved by the PMA
A civil suit filed against physician and/or
hospitals is premised on recovery of damages for
their wrongful act of employees.
CRIMINAL LIABILITIES Cause of action for damages is based on:
1. Breach of Contract
A criminal act is an outrage to the sovereignty of o Physician-patient relationship
the State so it must be instituted in the name of o Specific stipulation in the contract
the sovereign people as party-plaintiff (People of o In an action for breach of contract, the
the Philippines vs X) negligence of the doctor is not an issue,
Quantum of evidence is proof beyond for if the doctor makes contract to effect
reasonable doubt. This does not mean absolute a cure and fails to do so, he is liable for
certainty as excluding possibility of error but only breach of contract even though he uses
mean moral certainty, or that degree which the highest possible professional skill.
produces conviction in an unprejudiced mind 2. Tort (Quasi-delict)
(Rule 133, Sec. 2, Rules of Court) o Legal wrongdoing independent of a
Presumption of Innocence and Equipoise Rule contract
Conviction of a physician, aside from o Primary basis is negligence or fault of the
imprisonment and/or fine, his registration may physician as the one directly responsible
be cancelled or revoked if: for the injury sustained by the patient
a. The law imposes revocation of the license o Ordinarily, any malpractice action is
b. The crime wherein the physician was found based on torts or quasi-delicit in as much
guilty involved moral turpitude as negligence is usually a ground for
injury.
INCIDENTAL TO THE PRACTICE OF MEDICINE
Imprudence and Negligence Art. 365, Revised Art. 2176 of the Civil Code provides that
Penal Code Whoever by act or omission causes damage to
o Any person who, by reckless another, there being fault or negligence, is
imprudence, shall commit any act which obliged to pay for the damage done. Such fault
had it been intentional, would constitute or negligence, if there is no pre-existing
a grave felony, shall suffer the penalty of contractual relation between the parties, is
arresto mayor in its maximum period to called quasi-delicitxxx
prision correctional in its medium
period, if it would have constituted a less
grave felony, the penalty of arresto
mayor in its minimum period shall be MEDICAL MALPRACTICE
imposed; if it would have constituted a
light felony, the penalty of arresto Failure of a physician to properly perform the
menor shall be imposedxxx duty which devolves upon him in his professional
Imprudence deficiency of action or failing to relation to his patient which results to injury
take the necessary precaution once they are It may be defined as bad or unskillful practice of
foreseen medicine resulting to injury of the patient or
Reckless imprudence voluntary, without failure on the part of the physician to exercise
malice, doing or failing to do an act which results the degree of care, skill and diligence, as to
from material damage by reason of inexcusable treatment in a manner contrary to accepted
lack of precaution on the part of the person standards of medicine resulting to injury to the
performing the act, taking into consideration his patient
employment or occupation, degree of Elements:
intelligence, physical condition and other 1. The physician has a duty to the patient
circumstances regarding persons, time and 2. The physician failed to perform such duty to his
place. patient
Simple imprudence consist in the lack of 3. As a consequence of the failure, injury was
precaution displayed in those cases in which the sustained by the patient
damage impending to be caused is not 4. The failure of the physician is the proximate
immediate nor the danger clearly manifest cause of the injury sustained by the patient
Negligence indicates a deficiency of perception
or when the wrongful act maybe avoided by Proximate Cause is that cause, which, in
paying proper attention and using due diligence natural continuous sequence, unbroken by an
in foreseeing them. efficient intervening cause, produces the injury
and without which the result would not have DOCTRINE OF OSTENSIBLE AGENT
occurred In cases wherein the employees are at the same
1. There must be a direct physical connection time are independent contractors of the hospital
between the wrongful act of the physician and Because of this peculiar situation, they are
the injury sustained by the patient. considered ostensible agents and therefore, the
2. The cause or the wrongful act of the physician hospital must be held liable for their negligent
must be efficient and must not be too remote acts (pathologist, radiologist, anesthesiologist)
from the development of the injury suffered by
the patient. BORROWED SERVANT DOCTRINE
3. The result must be the natural continuous and Ordinarily, resident physicians, nurses and other
probable consequences. personnel of the hospital are employees or
servants of the hospital.
Doctrine of Efficient Intervening Cause In some instances, they are under the temporary
In causal connection between the negligence of supervision and control of another other than
the physician and the injury sustained by the their employer while performing their duties.
patient, there may be an efficient intervening By fiction of law, they are deemed borrowed
cause which is the proximate cause of the injury. from the hospital by someone and for any
wrongful act committed by them during the
period, their temporary employer must be held
liable for the discharge of their acts and duties.
LEGAL PRINCIPLES AND DOCTRINES In the determination whether one is a borrowed
APPLIED IN MEDICAL MALPRACTICE servant, it is necessary that he is not only
CASES subjected to the control of another with regard
to the work done and the manner of performing
Doctrine of Vicarious Liability it but also that the work to be done is for the
o Doctrine of Ostensible Agent benefit of the temporary employer.
o Borrowed Servant Doctrine
o Captain of the Ship Doctrine CAPTAIN-OF-THE-SHIP DOCTRINE
Doctrine of Res Ipsa Loquitor The doctrine enunciates liability of the surgeon
Doctrine of Common Knowledge not only for the wrongful acts of those who are
Doctrine of Contributory Negligence under his physical control but also those wherein
Doctrine of Assumption of Risk he has extension of control.
Doctrine of Last Clear Chance
Fellow Servant Doctrine REASONS FOR APPLICATION OF THE DOCTRINE OF
Rescue Doctrine VICARIOUS LIABILITY
1. Deep pocket theory
DOCTRINE OF VICARIOUS LIABILITY 2. The employer has the power to select his
Doctrine of Imputed Negligence/Command employee and to control his acts
Responsibility 3. Since the employer benefits monetarily from the
employee, the employer nor the employee is at
Vicarious liability means the responsibility of a
fault
person, who is not negligent, for the wrongful
4. To treat them as operating expense.
conduct or negligence of another
Art. 2180, Civil Code of the Philippines
DOCTRINE OF RES IPSA LOQUITOR
Obligation is demandable not only for ones own
The things speaks for itself, nature of the
acts or omission but also for those persons
wrongful act or injury is suggestive of negligence.
whom one is responsible
General rule: expert testimony is necessary to
xxx the owners or managers of an
prove that a physician has done a negligent act
establishment or enterprise are likewise
or that has deviated from the standard of
responsible for damages caused by their
medical practice
employees in the service of the branches in
Requisites of Res Ipsa Loquitor Doctrine:
which he latter are employed or on the occasion
1. The accident must of a kind which ordinarily
of their functions
does not occur in the absence of someones
Employers shall be liable for the damages caused
negligence.
by their employees and household helpers acting
2. It must be caused by an agency or
within the scope of their assigned task, even
instrumentality within the exclusive control of
though the former are not engaged in any
the defendant.
business or industry
3. It must not have been due to any voluntary
The responsibility treated of this article shall
action or contribution on the part of the plaintiff.
cease when the person herein mentioned prove
Some case wherein the Doctrine of Res Ipsa Loquitor has
that they observe all the diligence of a good
been applied:
father of a family to prevent injury.
1. Objects left in the patients body at the time of
caesarian section
2. Injury to a healthy part of the body may mitigate the damages to be
3. Removal of a wrong part of the body when awarded.
another part was intended Art 2214, Civil Code
4. Infection resulting from unsterilized instruments o In quasi-deficits, the contributory
5. Failure to take radiographs to diagnose a negligence of the plaintiff shall reduce
possible fracture the damages that he may recover.
Instances where the Doctrine of Res Ipsa Loquitor does o Some instances where there is
not apply: contributory negligence:
1. Where the Doctrine of Calculated Risk is 1. Failure to give the physician an
applicable accurate history
- When an accepted method of medical 2. Failure to follow the treatment
treatment involves hazards which may recommended by the physician
produce injurious results regardless of the 3. Leaving the hospital against the
care exercised by the physician advice of the physician
2. Bad Result Rule 4. Failure to seek further medical
3. Honest Errors of judgment as to Appropriate assistance if symptoms persist
Procedure Doctrine of Superior Knowledge
4. Mistake in the Diagnosis In the physician-patient relationship, the
physician has superior knowledge over his
In most medical malpractice suits, there is a patient. The patient just follows the instructions
necessity for a physician to give his expert and orders of the physician and is usually
medical opinion to prove whether acts of inactive and virtually places himself in the
omissions constitute medical negligence. This command and control of the physician.
doctrine has been regarded as rule of sympathy The defense of contributory negligence is
to counteract the conspiracy of silence. available only when the patients conduct is a
According to one of the most distinguished jurist truly flagrant disregard of his health and cannot
(Canada), Justice Mignault: apply where the patient is mentally ill,
o the practice of medicine and surery is semiconscious, heavily sedated or of advanced
indispensable to humanity and should age.
not be fettered by rules and
responsibility so strict as to exact an DOCTRINE OF CONTINUING NEGLIGENCE
infallibility on the part of the physician If the physician, after a prolonged treatment of a
which he does not possess. patient which normally produces alleviation of
o we would be doing a disservice to the the condition, fails to investigate non-response,
community at large if we were to impose he may be held liable if in the exercise of care
liabilities on hospitals and doctors for and diligence he could have discovered the
everything that goes wrong. Doctors cause of non-response.
would be led to think more of their own
safety than the good of the patients. DOCTRINE OF ASSUMPTION OF RISK
Initiative would be stiffed and Predicated upon knowledge and informed
confidence shaken. consent, anyone who voluntarily assumes the
risk of injury from a known danger, if injured, is
DOCTRINE OF CONTRIBUTORY NEGLIGENCE barred from recovery.
Doctrine of Common Fault violenti non fit injuria, which means that a
It has been defined as conduct on the part of the person who assents and was injured is no
plaintiff or injured party, contributing as a legal regarded in law to be injured
cause to the harm he has suffered, which falls
below the standard which he is required to DOCTRINE OF LAST CLEAR CHANCE
conform to his own protection. A physician who has the last clear chance of
It is the act or omission amounting to want of avoiding damage or injury but negligently fails to
care on the part of the complaining party which, do is liable.
concurring with the defendants negligence, is It implies thought, appreciation, mental
the proximate cause of the injury. direction and lapse of sufficient time to
Related Civil Code Provisions eventually act upon impulse to save the life or
Art. 2179, Civil Code prevent injury to another.
o When the plaintiffs own negligence
was the immediate and proximate cause DOCTRINE OF FORESEEABILITY
of his injury, he cannot recover A physician cannot be held accountable for
damages. But if his negligence was only negligence if the injury sustained by the patient
contributory, the immediate and is on account of unforeseen condition but if a
proximate cause of injury being the physician fails to ascertain the condition of the
defendants lack of due care, the plaintiff patient for want of the requisite skill and training
may recover damages, but the court
is answerable for the injury sustained by the 2. The relationship is terminated without mutual
patient if injury resulted thereto. consent of both parties
A physician owes duty of care to all persons who 3. Unilateral termination of the contractual
are foreseeably endangered by his conduct, with relationship by the physician
respect to the risk which make the conduct 4. Continuing need of the patient for further
unreasonably dangerous. medical treatment
5. Abandonment must have been the cause of the
FELLOW SERVANT DOCTRINE injury or death of the patient
This doctrine provides that if a servant Some instance of abandonment:
(employee) was injured on account of the 1. Refusal by a physician to treat a case after he has
negligence of his fellow servant (employee), the seen the patient needing medical treatment but
employer cannot be held liable. before treatment is commenced
2. Refusal to attend to a case for which he has
RESCUE DOCTRINE already assumed responsibility
If a physician who went to rescue a victim of an 3. Failure to provide follow-up attention
accident was himself injured, the original wrong 4. Failure to arrange for a substitute physician
doer must be held liable for such injury. during the time the physician is absent or
unavailable
SOLE RESPONSIBILITY VS SHARED RESPONSIBILITY o The attending physician may be held
Sole Responsibility liable for the acts of his substitute in the
When the negligent act or omission which is the following instances:
proximate cause of the injury suffered by patient a) The attending physician did
is attributed to the wrongful act of person. not exercise due care and
Shared Responsibility diligence in the selection of
When the injury suffered by the patient is caused the substitute
by the negligent act of two or more persons, b) If the substitute acts as agent
each of them acting concurrently and of the attending physician in
successively in the production of injury. so far as carrying out a certain
course of treatment in which
case master-servant
relationship is created
SPECIFIC ACTS OR OMISSIONS o N.B.
Non-payment of bill cannot be a
WHICH CONSTITUTE MEDICAL
defense for abandonment
MALPRACTICE
LIABILITIES IN THE ADMINISTRATION OF DRUGS
1. Failure to take medical history
Five basic rights:
2. Failure to examine or make a careful and
1. Right drug
adequate examination
2. Right patient
3. Non-referral of the patient to a specialist
3. Right dose
4. Failure to consult prior physicians for previous
4. Right time
management
5. Right route
5. Non-referral of patient to a hospital with
Negligence in the administration of a drug which causes
equipment and trained personnel
injury to the patient may be attributed to:
6. Failure to use the appropriate diagnostic test
1. Drug reaction
7. Failure to diagnose infections
- Failure to note history of allergy
8. Treatment resulting to addiction
- Failure to test for signs of reaction
9. Abandonment of patients
- Failure to stop treatment when the drug
10. Failure to give proper instructions
reaction has been observed
11. Failure to institute the proper prophylactic
- Failure to provide adequate therapy to
treatment
encounter a reaction
12. Errors in blood transfusion
- Treatment with a drug not proper for the
13. Liabilities in administration of drugs
illness
14. Product liabilities of manufacturer
2. Overdosage
15. Wrong baby cases
3. Failure to give warning of the side effects
4. Administering medicine on the wrong route
ABANDONMENT OF PATIENT
5. Administration of the wrong medicine
Termination of the physician-patient
6. Administration of a drug on the wrong person
relationship without the consent of the patient
7. Infection following an injection
and without giving the patient adequate notice
8. Injury to the nerves
and opportunity to find another physician
9. Failure to administer the drug
Elements:
1. There is a physician-patient relationship
DOCTRINE OF STRICT LIABILITY PRIMARY DUTIES OF A HOSPITAL
A person injured by a defective product can 1. To furnish a safe and well-maintained building
recover compensation from his injury from and ground
anyone in the distributive chain who sold the 2. To furnish adequate and safe equipment
product while the defect was present, even 3. To exercise reasonable care in the selection of
though the seller exercises every conceivable the hospital staff
caution to prevent and discover the defects. Persons coming within the premises of the hospital
Negligence or carefulness is not in issue in a case 1. Trespasser one who enters the property of
under the doctrine nor is any warranty or another without being granted the privilege to
promise in issue. A drug manufacturer is liable if do so. It is only required of a hospital to refrain
his product is contaminated by any impurities from taking positive steps to harm a trespasser.
which harm the user. 2. Licensee one who is neither a customer,
If the drug has side effects, it is the duty of the servant or a trespasser. He has no contractual
manufacturer to warn the physician of it either relation with the hospital. He is permitted,
through the literature attached or accompanying expressly or impliedly to be within the premises
the drug or through the services of the for his own interest or convenience. His
promoters. Once the physician has been presence is merely tolerated.
forewarned, the manufacturer has no duty to 3. Invitee one who is essential to the operation of
insure that the warning reaches the patient in a hospital or for whom the hospital has a
normal circumstances. purpose.

LIABILITIES OF HOSPITALS FOR THE WRONGFUL ACTS OF


THEIR AGENTS
LIABILITITES OF HOSPITALS 1. Government or Public Hospitals
- A State cannot be sued without its consent
Sec. 2(a), RA 4226, Hospital Licensure Act - The immunity of the government from the
HOSPITAL means a place devoted primarily to official acts of its officers, agents and
the maintenance and operation of facilities for employees is based on the legal principle
the diagnosis, treatment, and care of individuals that there can be no legal right against the
suffering from illness, disease, injury or authority that makes the law which the right
deformity, or in need of obstetrical or other depends.
medical and nursing care. The term hospital - RATIONALE: Government funds should be
shall also be construed as any institution, spent for public purposes and not diverted
building or place where there are installed beds, to compensate for private injuries and public
cribs, bassinets for twenty-four hour use or service should not be hindered. The
longer by patients in the treatment of government must not be sued because the
diseases,xxx. government derives no profit from its
Classification, according to Control and Financial activity unlike a private enterprise.
Support: - Those established to perform government
1. Public/Government operated and maintained functions, it is immune from being sued.
either partially or wholly by the national, - Those performing proprietary function when
provincial, municipal, or city government or it is established for profit. The government
other political subdivision, or by any goes down to the level of any private
department, division, board or other agency hospital.
thereof (Sec 2b, RA 4226) 2. Private Charitable, Voluntary or eleemosynary
2. Private privately owned, especially established for charity
and operated with funds raised and contributed - A charity hospital is established and
through donations, or private capital or other maintained from the donations,
means (Sec 2c, RA 4226) contributions, philanthropic acts and pays
For purposes of determining liability of private hospitals: no dividends.
1. Private charitable or eleemosynary established - The determination whether a hospital was
for the public benefit and not conducted for the established for charity is the articles of
pecuniary gain of the management incorporation and the constitution and by-
2. Private pay established for profit and gain laws of the corporation.
Rationale why hospital cannot practice medicine: 3. Private hospital operating for profit
1. The hospital cannot be subjected to government - May be held vicariously liable for the
licensure examinations to determine whether it negligent acts of its employees
is qualified to practice medicine
2. A non-medical will be allowed to control a N.B.
physician and through circumvention practice - A hospital which allow the patient to pay if
medicine ever they have the capacity to do so and
3. Breach of the confidential relationship in a serve others gratuitously does not change
physician-patient relationship the fundamental nature of the hospital as
charity.
- The charging of the fee is not controlling but negligent act was committed within the
the purpose the fee will be use is the scope of employment. But if entered with
measure of charity. the patient for contract of services, the
- The fact that a hospital refuses to accept principle independent contractor theory is
certain persons and others to pay in applied.
accordance with their means does not affect 4. Independent Contractor Theory
its charitable status, if it is operated for no 5. Sole Responsibility vs Shared Responsibility
profit.
- A charitable hospital must not consist of LIABILITIES OF HOSPITAL
rendering charitable acts to a few sporadic 1. CORPORATE Liabilities
cases but must be extended to the public - Those arising from failure of the hospital to
over a period of time. furnish accommodations and facilities
- A hospital established for profit even though necessary to carry out its purpose or to
some bed are devoted for charity is not follow in a given situation, the established
deemed a charitable institution. standard of conduct to which the
corporation should conform.
DOCTRINES APPLIED TO CHARITABLE HOSPITAL - Recent decisions of the court has extended
IMMUNITY FOR THE ACTS OF ITS EMPLOYEES hospital liability to patient for its failure to
Trust Fund Doctrine make careful selection, review, and
Charitable hospitals derived support from supervision of independent physicians who
voluntary contributions or donations for the are permitted to practice in the hospital
reception, care and treatment of charity 2. VICARIOUS Liabilities for the Acts of Hospital
patients. The contributions are held only in trust Employees
by the governing body of the hospital. Diverting Admission
the money for the payment of damage will be A person has no absolute right to be admitted in
utilizing the money not intended by the donor. a hospital or to avail of hospital services. The
Impaired Waiver Theory relationship between the hospital and the
A patient who enters a private hospital, knowing patient is contractual.
fully well that it is merely supported by A government has no absolute privilege of
contributions, waives his right to claim damages. choice of patients inasmuch as it is established
Public Policy Theory and maintained by public funds except for
It renders medical services without justifiable grounds.
remuneration. It is doing an undertaking of the Attendance to emergency cases in hospitals
obligation of State for the preservation of life Sec. 1 RA 6615 substantially states that xxx
and maintenance of health. hereby required to render immediate
Independent Contractor Theory emergency medical assistance and to provide
A patient who enters a private charitable facilities and medicine within its capabilities to
hospital does not have a contract with the patients in emergency cases who are in danger
hospital but with the attending physician. of dying and/or who may have suffered serious
injuries.
RULES APPLIED IN DETERMINING THE VICARIOUS Transfer of patients
LIABILITY FOR THE NEGLIGENT ACTS OF THE RESIDENT It must be premised on desire and consent of the
PHYSICIANS, NURSES AND OTHER EMPLOYEES patient and when the condition of the patient
1. Principle of administrative/ministerial as against would permit to do so.
professional/medical duties Discharge of patients
- The performance of all routinary duties After further evaluation of the patients
which is the very reason why he is appointed condition, considers that further hospitalization
in the ordinary sense constitutes is no longer indispensable, a physician may order
administrative duties and any negligent acts the discharge with or without condition.
committed by such employees in the course Refusal to be hospitalized
of their employment which causes injury, Refusal of the patient to remain in the hospital
the patient may make the hospital will not be a lawful ground to detain him if he is
vicariously liable. of sound mind and of legal age. Related laws:
- Medical duties are by its nature beyond the - Art. 268, Revised Penal Code
ordinary routine in a hospital. Any - 1987 Philippine Constitution, Sec. 1 and 6
negligence of such hospital employees, the Refusal of the patient to leave the hospital
borrowed servant doctrine must be Premature discharge
applied and the hospital may not be held The attending physician and the hospital any be
vicariously liable. held liable to the patient if the latter is discharge
2. Power of control from the hospital in spite of the fact that further
3. Contract of service hospitalization is still necessary.
- If the contract has been entered with Detention of patient for non-payment of bill
hospital to render professional services, the
hospital may be held liable provided the
A patient cannot be detained in a hospital for specialties, devices and poison are sold
non-payment of the hospital bill. The law at retail and where medical and dental
provides a remedy for them to pursue by filling veterinary prescriptions are
the necessary suit in court for the recovery of compounded and dispensed.
such fee or bill.
A hospital any legally detain a patient against his Medical Records
will when is detained or convicted prisoner, or It is compilation of the pertinent facts of the
when the patient is suffering from a very patients life history, illness, and treatment.
contagious disease wherein his release is It is a compilation of scientific data derived from
prejudicial to public health, or when the patient many sources, coordinated into a document and
is mentally ill, that his release will endanger made available for various uses to serve the
public safety. patient, the physician, the institution in which
the patient has been treated, the science of
medicine and society as a whole.
Purpose of Maintenance of Medical Records
LIABILITIES OF HOSPITAL FOR ITS 1. For convenience and necessity in consonance
ANCILLARY SERVICES with the purpose enumerated
2. As required by statutes (Hospital Licensure Law)
Whenever the hospital administration enters - The hospital may be held liable for injury
into contract with a partnership of physicians to resulting from a breach of duty to maintain
run the emergency room, the medical staff accurate records.
therein are not considered employees of the - Destruction of records is an evidence of
hospital. Consequently liability for negligence in negligence.
the emergency room is shifted to the medical - An altered medical record may create
partnership. suspicious intent to establish a defense and
Courts have held that even if contracts specify such alteration may be a proof of negligence.
that physicians will be considered independent - Removal of a certain portion of the record
contractors the hospitals are responsible for may raise the interference that they are
their action if they can exercise control over removed deliberately in order to suppress
them. evidence.
Patients are not bound by the secret limitations Ownership of Medical Records
contained in a private contract between the The guardian and owner of the medical records
hospital and the physician. is the hospital. But ownership of the medical
Two Aspects of Emergency Care record is a limited one and absolute and
1. Examination of the patient to determine his considered primarily custodial.
condition and need for emergency medical
procedures VIOLATION OF THE CONFIDENTIAL NATURE OF RECORD
2. Performance of the specific medical or surgical Sec. 6, Art. II, Code of Medical Ethics
procedure which are required without delay to The medical practitioner should guard as a
protect the patients health. sacred trust anything that is confidential or
Liability in the emergency room may arise from the private in nature that he may discover of that
following: may be communicated to him in his professional
1. Failure to admit relation with the patient, even after their death.
2. Failure to examine and/or treat He should never divulge this confidential
3. Negligence in the application of management information, or anything that may reflect upon
procedures the moral character of the person involved,
except when it is required in the interest of
An AMBULANCE is a motor vehicle specifically justice, public health or safety.
designed, equipped and used for the Sec. 17, Art. II, Comprehensive Dangerous Drug Act of
transportation of the sick, injured or wounded 2002 (Maintenance and Keeping of Original Records of
persons operated by trained personnel for Transactions on Dangerous Drugs and/or Controlled
ambulance service Precursors and Chemicals)
The criminal liability of an ambulance driver is The penalty of imprisonment ranging from 1 year
the same as that of an ordinary driver. However, to 6 years and a fine ranging from 10,000.00 to
the civil liability arising therefrom the hospital 50,000.00 shall be imposed upon any
must be held liable practitionerxxx who violates or fails to comply
with the maintenance and keeping of the original
Hospital Pharmacy records of transactions on any dangerous
Sec. 42, RA 5921 drugsxxx
o A PHARMACY is a place or establishment An additional penalty of revocation of license to
where drugs, chemical products, active practice his professionxxx
principles of drug, pharmaceuticals,
proprietary medicine of pharmaceutical
Sec. 60, Art. VIII, Comprehensive Dangerous Drug Act of therein stated, may be perceived as prima facie
2002 (Confidentiality of Records Under the Voluntary evidence, if such person made the entries in his
Submission Program) professional capacity or in the performance of
Judicial and medical records of drug dependents duty and in the ordinary course of business or
under the voluntary submission program shall be duty.
confidential and shall not be used against him for Patients record is admissible in evidence even if
any purposes, except to determine how many the person who made the entry is dead or not
time by himself or through his parent, spouse available, as the records are entries in the course
guardian or relative within the fourth degree of of business.
consanguinity or affinity, he voluntarily
submitted himself to confinement, treatment
and rehabilitation in any centerxxx
Sec. 64, Art. VIII, Comprehensive Dangerous Drug Act of DAMAGES
2002 (Confidentiality of Records Under the Comulsory
Submission Program) DAMAGES are the pecuniary compensations
The records of a drug dependent who was that may be recovered for breach of some duty
rehabilitated and discharged from the Center or the violation of some rights recognized by law.
under the compulsory submission program or If a suit is filed against a physician for a
who was charged for violation of Sec. 15 of this professional liability claims, the objective of the
act shall be covered under Sec. 60 of this act. plaintiff is to recover damages. If the physician is
However, the records of a drug dependent who found negligent in the performance of his
was not rehabilitated, or who has escaped but professional services, he is liable for the payment
did not surrender himself within the prescribed of damages for all the direct, natural and logical
period, shall be forwarded to the court and their consequences of his act.
use shall be determined by the court, taking into Art. 20, Civil Code
consideration public interest and the welfare of Every person, contrary to law, willfully or
the drug-dependent. negligently causes damage to another shall
Sec. 21(c), Rule 130, Rules of Court (Privilege indemnify the latter for the same.
Communication) Art. 2176, Civil Code
A person authorized to practice medicine, Whoever, by act or omission causes damage to
surgery, obstetrics cannot in civil case, without another, there being fault or negligence, is
the consent of the patient, be examined as to obliged to pay for the damages done.
any information, which he may have acquired in
attending such patient in a professional capacity, Damages must be sufficiently proven by
which information was necessary to enable him evidence. To permit the Court to determine how
to act in that capacity, and which would blacken much it must, the proof must show the nature,
the character of the patient. extent, cause and probably duration of the
injury.
Information for which no authorization is needed
1. Name of the patient and house officers DOCTRINE OF CERTAINTY OF DAMAGES
associated with the treatment of a patient Damages must be certain both in its nature and
2. Personal circumstances of the patient which are in respect to the cause. Recovery must not be
not ordinarily related to the treatment contingent or speculative.
The injured patient has the right to recover
When may the contents of the record be disclosed medical and hospital expenses from the
1. When requested by the patient or by someone wrongdoer even though the patient has been
who could act in his behalf which must be made indemnified wholly or partially by an insurance
in writing company.
2. When the law requires such disclosure
3. Upon a lawful order of the court TYPES OF DAMAGES
N.B. 1. Actual or compensatory
The attending patient has no legal right to 2. Moral
determine who shall and who shall not see the 3. Exemplary
record. At the most, his approval or permission 4. Nomina
is only a matter of courtesy. 5. Temperate
Members of the resident staff, student and 6. Liquidated
attending medical staff may freely consult such
records as pertain to their work. ACTUAL OR COMPENSATORY DAMAGES
Sec. 37, Rule 130, Rules of Court Entries in the Course Art. 2199, Civil Code
of Business Except as provided by law or by stipulation, one
Entries made at, or near the time of the is entitled to an adequate compensation only for
transactions to which they refer, by a person such pecuniary loss suffered by him as he has
deceased, outside the Philippines or unable to
testify, who was in a position to know the facts
duly proved. Such compensation is referred to as Art. 2231, Civil Code
actual or compensatory damages. In quasi-delict, exemplary damages may be
Art. 2200, Civil Code granted if the defendant acted with gross
Indemnification for damages shall comprehend negligence.
not only the value of the loss suffered, but also Art. 2232, Civil Code
that of the profits which the oblige failed to In contracts and quasi-contracts, the court may
obtain award exemplary damages if the defendant
Kinds: acted in a wanton, fraudulent, reckless,
1. Dano emergente the loss already suffered by oppressive and malevolent manner.
the patient Art. 2233, Civil Code
2. Lucro cesante failure to receive the benefit Exemplary damages cannot be recovered as a
which would have pertained to him matter of right; the course will decide whether
Compensatory Damages Applied to Medical Malpractice or not they should be adjudicated
1. Death
- Art. 2206, Civil Code Punitive or exemplary damages are monetary
o The amount of damages for the compensation over and above actual or
death caused by a crime or quasi- compensatory damages awarded as punishment
delict shall be at least three or deterrence, because of the wanton, reckless,
thousand pesos (75,000.00), even malicious or oppressive nature of the wrong
though there may have been committed.
mitigating circumstance. Punitive damages are recoverable from a
2. Physical Disability physician in an action for malpractice where
3. Loss of Earning Capacity there is evidence tending to show that he has
4. Medical, Surgical, Hospital and Related Expenses acted with malice, or that he acted with
5. Loss of Service or Support recklessness, oppression, or with utter disregard
6. Funeral Expenses to the effects of his act, or that he is guilty of
gross negligence in the performance of his
MORAL DAMAGES profession.
Art. 2217, Civil Code
Moral damages include physical suffering, NOMINAL DAMAGES
mental anguish, fright, serious anxiety, Art. 2221, Civil Code
besmirched reputation, wounded feelings, Nominal damages are adjudicated in order that
moral shock, social humiliation, and similar a right of the plaintiff, which has been violated
injury. Through incapable of pecuniary or invaded by the defendant, may be vindicated
computation, moral damages may be recovered or recognized not for the purpose of
if they are the proximate result of the indemnifying the plaintiff for any loss suffered.
defendants act or omission. It is a trifling sum awarded to the plaintiff in an
Art. 2219, Civil Code action where there is no substantial loss or injury
Moral damages may be recovered in the to be compensated.
following and analogous cases: It is awarded to plaintiff as a vindication of a right
1. A criminal offense resulting in physical violated.
injuries
2. Quasi-delict causing physical injuries LIQUIDATED DAMAGES
Physical suffering Art. 2226, Civil Code
Mental Anguish Liquidated damages are those agreed upon by
Fright and Moral Shock the parties to a contract, to be paid in case of
Besmirched Reputation and Social breach thereof.

EXEMPLARY OR CORRECTIVE DAMAGES TEMPERATE OR MODERATE DAMAGES


Art. 2229, Civil Code Art. 2224, Civil Code
Exemplary or corrective damages are imposed, Temperate or moderate damages, which are
by way of example or correction for the public more than nominal but less than compensatory
good, in addition to the moral, temperate, damages, may be recovered when the court
liquidated or compensatory damages. finds that some pecuniary loss has been suffered
Art. 2230, Civil Code but its amount cannot, from the nature of the
In criminal offense, exemplary damages as part case, be proved with certainty.
of the civil liability may be imposed when the
crime was committed with one or more Obligation on the part of the plaintiff (patient) to
aggravating circumstances. Such damages are minimize damages
separate and distinct from fines and shall be paid Art. 2203, Civil Code
to the offended party. o The party suffering from loss or injury
must exercise the diligence of a good
father of a family to minimize the
damages resulting from the act or professional conscience. He should, however,
omission in question. always respond to any request for his assistance
in an emergencyxxx
DOCTRINE OF AVOIDABLE CONSEQUENCES Art. II, Sec. 3
Where one person has, through wrongful act, In case of emergency, wherein immediate
caused personal injury to another, it is action is necessary, a physician should
incumbent upon the latter to use such means as administer at least first aid treatment and then
are reasonable under the circumstances to avoid refer the patient to a more qualified and
or minimize the damages. The person wronged competent physician if the case does not fall
cannot recover for any item of damage which within his particular line.
could have been avoided. Art. IV, Sec. 15
The burden of proof that the injured could have A physician should never examine or treat a
prevented or mitigated the damages rests on the hospitalized patient of another without the
defendant. latters knowledge and consent except in cases
of emergencyxxx
ATTORNEYS FEES Art. IV, Sec. 16
Attorneys fees and other related expenses in litigation, A physician called upon to attend to a patient of
other than judicial cost are not as rule recoverable except another physician because of an
when the law specifically provides. (Art. 2208, Civil Code) emergencyxxxshould attend only to the
1. When exemplary damages are awarded patients immediate needsxxx
2. When the defendants act or omission has Art. IV, Sec. 17
compelled the plaintiff to litigate with third Whenever in the absence of the family
persons or to incur expenses to protect his physician several physicians have been
interest simultaneously called in an emergency
3. In criminal cases of malicious prosecution casexxxthe first to arrive should be
against the plaintiff considered physician in charge, unless the
4. In case of clearly unfounded civil action or patient or his family has special preference for
proceeding against the plaintiff some other one among those who are
5. Where the defendant acted in gross and evident presentxxx
bad faith in refusing to satisfy the plaintiffs Art. IV, Sec. 20
plainly valid, just and demandable claim When a physician is requested by a colleague to
6. In actions for legal support take care of a patient because of an
7. In actions for the recovery of wages of household emergencyxxx The physician should treat the
helpers, laborer and skilled workers patient in the same manner and with the same
8. In actions for indemnity under the workmens delicacy as he would have wanted his own
compensation and employers liability laws patient cared for under similar conditionsxxx
9. In a separate civil action to recover civil liability
10. When at least double judicial costs are awarded OTHER RELATED PROVISIONS OF THE LAW
11. In any other case where the court deems it just RA 6615
and equitable that attorneys fees and expenses An act requiring government and private
of litigation should be recovered hospitals or clinics duly licensed to extend
medical assistance in emergency cases
RA 8344
An act penalizing the refusal of hospitals and
EMERGENCIES IN MEDICAL medical clinics to administer appropriate initial
PRACTICE medical treatment and support in emergency or
serious cases, amending BP Blg. 702, otherwise
An EMERGENCY is an unforeseen combination of known as an act prohibiting the demand or
circumstances which calls or an immediate deposits or advance payments for the
action. It refers to a situation in which a patient confinement or treatment of patients in hospital
has been suddenly or unexpectedly endangered and medical clinics in certain cases.
to such an extent that immediate action is Art. 275, Revised Penal Code
needed to save the life and limb or to avoid Abandonment of persons in danger and
permanent damages. abandonment of ones own victim
The penalty of arrest mayor shall be imposed
RELATED PROVISION OF THE CODE OF MEDICAL ETHICS upon:
A condition of emergency is usually an exception 1. Anyone who shall fail to render assistance to
to the observance of the standard ethical any person whom he shall find in an
conducts. uninhabited place wounded or in danger or
Art. II, Sec. 2 dying, when he can render such assistance
A physician is free to choose whom he will without detriment to himself, unless such
serve. He may refuse calls, other medical omission shall constitute a more serious
services for reasons satisfactory to his offense
2. Anyone who shall fail to help or render agreed upon, the principle of quantum meruit
assistance to another whom he has shall be applied.
accidentally wounded or injured

STANDARD CARE IN EMERGENCIES


A physician cannot be held to the same conduct DELEGATION OF A PHYSICIANS
as one who had an opportunity to reflect, even DUTIES
though it later appears that he made a wrong
decision yet prudent at that time. Requisites for a Valid Delegation
1. When such duty can be delegated which will
EMERGENCY OPERATIONS WITHOUT CONCENT depend on the circumstance of the case, nature
When the situation is such that an immediate of the duty to be delegated, and the training and
action is necessary to save the life or preserve experience of the person to whom such duty is
the health of the patient, and getting a consent to be delegated
is prejudicial to the patient, the physician can 2. The person to whom such duty is delegated must
legally proceed with his contemplated life-saving be competent to perform such duty
procedure. 3. Proper instructions must be given to the person
The law gives him the right to act under the who will perform the delegated duty
Theory of Implied Consent or that the physician 4. The patient consented expressly or impliedly
is privileged to do whatever is sound for the such delegation of duty
benefit of the patient.
The refusal f the patient who is of legal age and LIABILITY FOR INJURIES IN THE NEGLIGENT
of sound mind to submit to medical treatment PERFORMANCE OF THE DELEGATED DUTIES
shall prevail even if the danger to his life is The person performing the delegated duty
eminent. cannot be held liable for any untoward or
unexpected effects of his act if he had complied
EMERGENCY OPERATION WITHOUT CONSENT with all the requirements of a delegated duty
1. The injured person must be unconscious or and has exercised care and diligence in such
otherwise unable to give a valid consent execution.
2. The situation must be such as it would make it
actually and apparently necessary to act before
there is an opportunity to obtain consent.
3. The physician in the exercise of his best THE MEDICAL WITNESS AND THE
judgment that the medical procedure is life- COURT
saving.
Court an agency of the sovereign created
REFUSAL TO GIVE CONSENT DURING EMERGENCY directly or indirectly under its authority,
IF the patient is conscious, or if unconscious or is constituting one or more officers, established
not in a capacity to give consent, but someone and maintained for the purpose of hearing and
who could act on his behalf is present, then determining issues of law and facts regarding
consent must first be obtained before the legal rights and alleged violations thereof, and of
commencement of a procedure. applying the sanction of the law, authorized to
exercise its power in the due course of law at
EXTENSION OF OPERATION IN CASES OF EMERGENCY times and places previously determined by
If during an operation, an accident occurs, or a lawful authority
condition maybe discovered which requires Different Courts in the Philippines
immediate action, but which is not covered by 1. Supreme Court
the consent, the surgeon is justified in extending 2. Court of Appeals
the operation and be absolved of liability. 3. Regional Trial Court
The surgeon is authorized to extend the 4. Municipal or City Trial Court
operation to any condition discovered when it 5. Military Commissions
will redound to the welfare of the patient. In the Art. III, Sec. 2, Code of Medical Ethics
absence of a clearly specific prohibition on the It is the duty of every physician, when called
part of the patient, the physician should be upon by the judicial authorities, to assist in the
privileged to perform such surgery within the administration of justice on matters which are
operative field as is justified in the prevailing medico-legal in character.
medical opinion.
N.B. PHYSICIAN AS AN ORDINARYY WITNESS
Surgery cannot be extended if an emergency is Sec. 20, Rule 130, Rules of Court
not present. xxxall persons who, having organs of sense,
The law also implies an obligation on the part of can perceive, and perceiving can make known
the patient to pay reasonable value of the their perception to others maybe witnessxxx
emergency service. If no specified amount
Requisites of an Ordinary Witness with a view to professional employment; nor
1. The person must have the organ and power of can an attorneys secretary, stenographer of
perception clerk be examined, without the consent of
2. The perception gathered by his organs of sense the client and his employer, concerning any
can be imparted to other fact the knowledge of which has been
3. He does not fall in any of the exception or acquired in such capacity
disqualifications provided by the Rules of Court 3. A person authorized to practice medicine,
Disqualification by reason of: surgery or obstetrics cannot in a civil case,
1. Mental incapacity or immaturity Sec. 21, Rule without the consent of the patient, be
130, Rules of Court examined as to any information which he
a. Those whose mental condition, at the may have acquired in attending such patient
time of their production for in a professional capacity, which information
examination, is such that they are was necessary to enable him to act in that
incapable of intelligently making known capacity, and which would blacken the
their perception to others reputation of the patient
b. Children whose mental maturity is such 4. A minister or a priest cannot, without the
as to render them incapable of consent of the person making the
perceiving the facts respecting which confession, be examined as to any
where they are examined and of relating confession made to or any advice given him
them truthfully in his professional character in the course of
2. Marriage Sec. 22, Rule 130, Rules of Court discipline enjoined by the church to which he
- During their marriage, neither the husband belongs
nor the wife may testify for or against the 5. A public officer cannot be examined during
other without the consent of the affected his term of office or afterwards as to
spouse, except in a civil case by one against communications made to him in official
the other, or in any criminal case for a crime confidence, when the court finds that the
committed by one against the other or the public interest would suffer by the disclosure
latters direct ascendants or descendants
3. Death or Insanity Sec. 23, Rule 130, Rules of PRIVILEGED COMMUNICATION BETWEEN PHYSICIAN
Court AND HIS PATIENT
- Parties or assignors of parties to a case, or Sec. 6, Art. II, Code of Medical Ethics
persons in whose behalf a case is The medical practitioner should guard as sacred
prosecuted, against an executor or trust anything that is confidential or private in
administrator or other representative of the nature that he may discover or that may be
deceased person, or against a person pf communicated to him in his professional relation
unsound mind, upon a claim or demand with his patients, even after their death. He
against the estate of such deceased person should never divulge this confidential
or against such person of unsound mind, information, or anything that may reflect upon
cannot testify as to any matter of fact the moral character of the person involved,
occurring before the death of such deceased except when it is required in the interest of
person or before such person become of justice, public health, and public safety.
unsound mind Sec. 24(c), Rule 130, Rules of Court
4. Parental or filial privilege Sec. 25, Rule 130, A person authorized to practice medicine,
Rules of Court surgery or obstetrics cannot in a civil case,
- No descendant can be compelled, in a without the consent of the patient, be examined
criminal case, to testify against his parents as to any information which he may have
and descendant acquired in attending such patient in a
Disqualification by reason of Privileged Communication professional capacity, which information was
Sec. 24, Rule 130, Rules of Court The following necessary to enable him to act in that capacity,
persons cannot testify as to matters learned in and which would blacken the character of the
confidence of the following cases: patient.
1. The husband or the wife, during or after the Extent of the Privileged Communication
marriage, cannot be examined without the 1. Interns
consent of the other as to any 2. Confidential information obtained by one of the
communication received in confidence by physicians practicing medicine in partnership
one from the other during the marriage with another physician whereby the patients of
except in a civil case by one against the both are the patients of the firm.
other, or for a crime committed by one 3. Nurses and attendants who were present and
against the other or the latters direct assisting the physician when the communication
descendants and ascendants was made.
2. The attorney cannot, without the consent of When communication is not a privileged communication
his client, be examined as to any 1. When a person is examined at the instance of the
communication made by the client to him or law, for the purpose of testifying to solely qualify
his advice given thereon in the course of, or the physician to testify
2. Information acquired by an autopsy on the body 2. That at the time the declaration was made, the
of a person who was not, prior to his death, a declarant was under the consciousness of
patient of the physician performing the autopsy impeding death
3. Information obtained by a technician from a 3. That the declarant is a competent witness
patient is not privileged. 4. That the declaration is offered in a criminal case
4. When the public interest so requires in which the declarant is the victim
Scope of the privilege Probative value of standard medical books in court
1. Oral testimony by the physician in court Medical textbooks are not admissible in
2. Affidavits, certificates and reports made by the evidence on account of the fact that they are
physician as exhibit in court hearsay
3. Hospital records The author of the books cannot be presented in
Waiver of Privilege court and be subjected to cross-examination
Patient may expressly or impliedly waive their N.B.
right to privilege communication o Medical witness may have the right to
base his opinion from standard
HEARSAY EVIDENCE textbooks.
An evidence not proceeding from personal o In cross-examination, medical textbooks
knowledge of the witness, but from mere are admissible in evidence to discredit a
repetition of what he had heard others say. It witness who has based his testimony
does not derive its value solely from the credit of upon it.
the witness but its value rests mainly in the Learned Treaties
veracity and competency of other persons. The Sec. 46, Rule 130, Rules of Court
very nature of the evidence shows its weakness, o A published treatise, periodical or
and it is admitted only in special cases because pamphlet on a subject of history, science
of necessity. or art is admissible as tending to prove
As a general rule, it is NOT admissible in evidence the truth of a matter stated therein if the
Sec. 36, Rule 130, Rules of Court Testimony generally court takes judicial notice, or a witness
confined to the personal knowledge of the witness, expert in the subject testifies that the
hearsay excluded writer of the statement in the treatise,
A witness can testify only to those facts which periodical or pamphlet is recognized in
he knows of his own knowledge, that is, which is his profession or calling as expert in the
derived from his own perception, except as subject.
otherwise provided by these rules.
Sec. 37, Rule 130, Rules of Court THE PHYSICIAN AS AN EXPERT WITNESS
The declaration of a dying person, made under An EXPERT WITNESS is on who has the capacity
a consciousness of an impending death, may be to draw inference from the facts which a court
received in a criminal case wherein his death, would not be competent to draw.
may be received in a criminal case where his To warrant the use of expert testimony, two
death is the subject of inquiry, as evidence of the elements are required:
cause and surrounding circumstances of such 1. The subject of inference must be so distinctly
death. related to some science, profession,
Dying declaration business or occupation as to be beyond the
One of the exception in hearsay evidence rule. knowledge of average layman
Is the statement made by a person who is at the 2. The witness must have such skill, knowledge
point of death, and is conscious of his impending or experience in that field or calling as to
death, in reference to the manner in which he make it appear that his opinion or reference
receives his injuries of which he is dying, or the will probably aid the trier of facts in his
immediate cause of his death, and in reference search for the truth
to the person who inflicted such injuries or in An OPINION maybe defined as the belief,
connection with such injuries of a person who is judgment, inference, or sentiment formed by the
charged or suspected of having committed mind with regard to things, person or events
them; which statements are admissible in perceived by a witness. An opinion in the legal
evidence in a trial where the killing of the sense is something more than mere speculation
declarant is the crime charged to the defendant. or conjecture.
Grounds for admissibility of dying declaration Sec. 48, Rule 130, Rules of Court
1. Necessity death of the declarant makes it General rule The opinion of a witness is not
impossible to obtain his testimony in court admissible, except as indicated by the Rules
2. Trustworthiness every motive of falsehood is Sec. 49, Rule 130, Rules of Court
silenced, and the mind is induced by the most Opinion of Expert Witness The opinion of a
powerful consideration to tell the truth witness on a matter requiring special knowledge,
Requisites of dying declaration skill, experience or training which he is shown to
1. That the declaration must concern the cause and possess, may be received in evidence
surrounding circumstances of the declarants
death
Sec. 50, Rule 130, Rules of Court Sec. 24, Art. III, Medical Act of 1959 as amended
Opinion of Ordinary Witness The opinion of a xxx(12) Violation of any of the provisions of
witness for which proper basis is given, may be the Code of Medical Ethicsshall be sufficient
received in evidence regarding ground for reprimanding or for suspending or
a. The identity of a person about whom he revoking a certificate of registration as
has adequate knowledge physicianxxx
b. A handwriting with which he has Refusal of a public officer to give assistance in
sufficient familiarity the administration of justice is penalized by law.
c. The mental sanity of a person with Art. 233, Revised Penal Code
whom he is sufficiently acquainted The penalties of arresto mayorxxx, shall be
The witness may also testify on his impression of imposed upon a public officer who, upon
emotion, behavior, condition or appearance of a demand from a competent authority, shall fail to
person. lend his cooperation towards the administration
of justice or other public service, if such failure
DISTINCTIONS BETWEEN AN ORDINARY WITNESS AND shall result in serious damage to the public
EXPERT WITNESS interest, or to a third party.
1. An ORDINARY witness can only testify as a When a Medical Witness need not comply with a
general rule, on those things which he has subpoena
perceived with his own organs of perception, 1. The Court issuing the subpoena has no
while an EXPERT witness may render his opinion, jurisdiction over the subject matter of the case
inference, conclusion or deduction on what he 2. When the place of residence is more than 100
and other perceived kilometers from the court issuing the subpoena
2. An ORDINARY witness need not be skilled on the - Sec. 10, Art. 21, Rules of Court
line he is testifying but an EXPERT witness must o The provisions of sections 8 and 9
be skilled on the art, science or trade he is of this rule (Compelling attendance
testifying and Contempt) shall not apply to a
witness who resided more than 100
ATTENDANCE OF A MEDICAL WITNESS IN COURT km from his residence to the place
Sec. 1, Rule 21, Rules of Court where he is to testify by the ordinary
Subpoena is a process directed to a person course of travel, or to a detention
requiring him to attend and to testify at the prisoner if no permission of the cut
hearing or the trial of an action, or at any in which his case is pending was
investigation conducted by competent obtained.
authority, or for the taking of his deposition. It 3. When the physician is attending to an
may also require him to bring with him books, emergency and no one is available and
documents or other things under his control, in competent enough to be his substitute to attend
which case it is called subpoena duces tecum. to such emergency
Kinds of Subpoena 4. On account of illness incapacitating him to
1. Subpoena ad testificandum attend
- A process requiring a person to appear Sec. 3, Rule 132, Rules of Court, Rights and Obligations of
before a trial or hearing of an action or a Witness
investigation conducted under our laws or A witness must answer questions, although his
for the taking of a deposition at a certain answers may tend to establish a claim against
definite date, time and place to testify on him. However, it is the right of witness:
some material issues. 1. To be protected from irrelevant, improper
2. Subpoena duces tecum questions and from harsh or insulting
- A process which requires a person to demeanor
produce at the trial some documents or 2. Not to be detained longer than the interest
papers which are under his control or of justice require
possession that are pertinent to the issues of 3. Not to be examined except only as to
his controversy, at a certain date, time and matters pertinent to the issue
place. 4. Not to give an answer which will tent to
3. Subpoena duces tecum and testificandum subject him to a penalty for an offense
N.B. unless otherwise provided by law
Failure to comply with a subpoena without 5. Not to give an answer which will tend to
justifiable reason is a ground for reprimand, degrade his reputation, unless it be to the
suspension or revocation of the certificate of very fact at issue or to a fact from which the
registration. fact in issue would be presumed. But a
Sec. 2, Art. III, Code of Medical Ethics witness must answer to the fact of his
It is the duty of every physician, when called previous final conviction of an offense.
upon by the judicial authorities, to assist in the Instances when the medical witness may not be
administration of justice on matters which are compelled to answer questions in court
medico-legal in character. 1. It will tend to subject him to punishment for an
offense
- Incriminatory questions may subject the 4. Art. 183, Revised Penal Code, False testimony in
witness to punishment or disclosure of other cases and perjury in solemn affirmation
which would form a necessary and essential - The penalty of arresto mayorxxx,
part of a crime knowingly making untruthful statement and
- Sec. 17, Art III, Phil Constitution No person not being included in the provisions of the
shall be compelled to be witness against next preceding articles, shall testify under
himself. oath, or make an affidavit, upon ant material
2. It will degrade his character, except when such before a competent person authorized to
degradation of character is the very fact at issue. administer an oath in cases in which the law
so requires.
A medical witness may refer to memoranda, - Any person, who in case of a solemn
notes or other pertinent papers. affirmation made in lieu of an oath, shall
commit any of the falsehoods mentioned in
IMPEACHMENT OF MEDICAL TESTIMONY this and the three preceding articles of this
1. By contradicting testimonies by other of his own section, shall suffer the respective penalties
class or by any other competent witnesses provided therein.
2. By showing that the medical witness is
interested in the outcome of the case or bias PRINCIPLE OF Falsus In Uno, Falsu In Omnibus
3. By an inconsistent statement made at another When a witness falsified the truth on one point,
time his testimony on other points may be
4. By not expressing the opinion testified to at the disregarded, unless corroborated by other
time when such expression might reasonably unimpeached evidence.
had been expected Requisites:
5. When the scientific treatise which he relies on as 1. That the witness deliberately or intentionally
the basis of his opinion does not sustain him falsified the truth
2. That the other portions of the testimony to be
EFFECTS OF FALSE TESTIMONY discredited, are not corroborated by
1. Art. 180, Revised Penal Code, False testimony circumstance or other unimpeached evidence
against a defendant: 3. The false testimony must be on material pot
- Any person who shall give testimony When it is not applicable
against the defendant in any criminal case 1. When there are sufficient corroborations in
shall suffer: many grounds of the testimony
i. The penalty of reclusion temporal, if 2. When the mistake was not in a very material
the defendant in said case shall have point
been sentenced to death 3. When the error did not arise from the apparent
ii. The penalty of prision mayor, if the desire to prevent the truth, but from innocent
defendant shall have been mistakes and the desire of the witness to
sentenced to reclusion temporal or exculpate himself though not completely
perpetua
iii. The penalty of prision correccional, CONTEMPT
if the defendant shall have been Sec. 9, Rule 21, Rules of Court
sentenced to any other afflictive Contempt Failure by any person without
penalty adequate cause to obey a subpoena served upon
iv. The penalty of arresto mayor, if the him shall be deemed a contempt of court from
defendant shall have been which the subpoena is issued. If the subpoena
sentenced to a correccional penalty was not issued by a court, the disobedience
or a fine, or shall have been thereto shall be punished in accordance with the
acquitted. applicable law or Rule.
2. Art. 181, Revised Penal Code, False Testimony Sec. 1, Rule 71, Rules of Court
favorable to a defendant Direct Contempt Punished Summarily A
- Any person who shall give false testimony person guilty of misbehavior in the presence of
in favor of the defendant in a criminal case, or so near a court as to obstruct or interrupt the
shall suffer the penalties of arresto mayor in proceedings before the same, including
its maximum period to prision disrespect toward the court, offensive
correccionalxxxif the prosecution is for personalities toward others, refusal to be sworn
felony punishable by an afflictive or to answer as witness, or to subscribe an
penaltyxxx. affidavit or deposition when lawfully required to
3. Art. 182, Revised Penal Code, False Testimony in do so, may be summarily adjudged in contempt
Civil Cases by such courtxxx
- Any person found guilty of false testimony Sec. 3, Rule 71, Rules of Court
in civil cases shall suffer the penalty of Indirect contempt to be punished after charged
prision correcccionalxxx if the amount in and hearing After a charge in writing has been
controversy shall exceed 5,000 pesosxxx filed, and an opportunity given to the
respondent to comment thereon within such
periodxxx, a person guilty of any of the friend of man
following acts may be punished for indirect Art. II Duties of Physicians to their Patients
contempt: To attend to his patient faithfully and
1. Misbehavior of an officer of a court in the conscientiously
performance of his official duties or in his Free to choose whom to serve
official transaction Immediate action in cases of emergency
2. Disobedience of or resistance to a lawful Proper consultation and referral
writ, process, order, or judgment of a court He must exercise good faith and strict honesty in
3. Any abuse of or any unlawful interference expressing his opinion
with the process or proceedings of a court Sacred trust of information
not constituting direct contempt Practice of medicine is not a business
4. Any improper conduct tending directly or Art. III Duties of Physicians to the Community
indirectly to impede, obstruct, or degrade Cooperation with proper authorities, sanitation
the administration of justice and health
5. Assuming to be an attorney
To assist in the administration of justice
6. The rescue of a person or property in the
To protect the public from charlatans
custody of an officer by virtue of an order
No solicitation and extravagant and false
But nothing in this section shall be construed as
advertisement
to prevent the court from issuing process to
Gratuitous services to the indigents
bring the respondent to the court, or from
Art. IV Duties of Physicians to their Colleagues and to
holding him in custody pending such
the Profession
proceedings.
Gratuitous services to a colleague
HYPOTHETICAL QUESTIONS PROPUNDED TO A MEDICAL Proper consultation and referral
WITNESS Uphold the honor and dignity of the profession
A physician may be requested to draw his Observe punctuality
conclusion or opinions from asset of facts which Observe utmost caution, tract and prudence as
may or may not be known to him. He must regards professional conduct of another
answer the question provided those set of facts physician
has the following requisites: To refrain from making unfair and unwarranted
1. It must be framed as to fairly represent those criticisms of other physicians
facts and not give a situation a false color by To keep abreast to the advancement of medical
the way the statement was given. science and contribute to its progress
2. The hypothetical questions is based upon He should be diligent, upright, sober, modest
facts which are in evidence and assumed and well-versed in both the science and art of
facts within the limit of evidence. medicine
3. The hypothetical question must not be No advertising by means of untruthful or
unfair or misleading. improbable statements in newspapers or
Where the expert is familiar with the facts by exaggerated announcements
personal observation and so testifies, he may be Should expose without fear or favor, before the
asked directly for his opinion, without stating the proper medical or legal tribunals corrupt and
facts upon which it is based. dishonest conduct of members of the profession
When the expert is not familiar with the facts Should aid in safeguarding against the admission
upon which it is based, they must be stated to of those who are unfit or unqualified because of
him hypothetically, and upon the assumption of deficiency in moral character
the facts so stated, he must base his opinion. Art V Duties of Physicians to Allied Professionals
To cooperate with and safeguard the interest,
RATIONALE OF THE CORROBORATIVE PROBATIVE VALUE reputation and dignity of allied professional
OF MEDICAL EXPERT TESTIMONIES Not to allow to be published any testimonial
1. The testimonies however impartial, may certifying efficacy, value and superiority and
unconsciously favor one of the part litigants. The recommendation of drugs
human mind is not absolutely impossible. Non-payment of commissions to an person who
2. The opinion of the medical expert is based on refers cases to help him acquire patients
experience and treatise or books which may be Penal Provision
divergent from the observation of others. Violation of the provisions of this Code
constitute unethical and unprofessional conduct
and therefore a sufficient ground for the
reprimand, suspension or revocation of the
CODE OF MEDICAL ETHICS certificate of registration of the offending
physician in accordance with the provisions of
Art. I General Principles Sec. 24, par. 12 of the Medical Act of 1959
Primary objective of the practice of medicine is
SERVICE to mankind
RIGHTS AGAINST SELF-INCRIMINATION
Sec. 17, Art. III, Philippine Constitution states that,
No person shall be compelled to witness against
himself.

The right is available not only in criminal


prosecutions but also in all other government
proceedings, including civil actions and
administrative or legislative investigations. It
may be claimed not only by the accused but also
by any witness to whom a questions calling for
an incriminating answer is addressed.
In criminal actions, the accused may not be
compelled to take the witness stand, on the
reasonable assumption that the purpose of the
interrogation will be to incriminate him.
The same principle shall apply to the respondent
in an administrative proceeding where the
respondent may be subjected to sanctions of a
penal character, such as cancellation of his
license to practice medicine (Pascual vs. Board of
Medical Examiners, 28 SCRA 345)
Scope:
The kernel of the right is not against all
compulsion, but testimonial compulsion only.
It is simply against the legal process of extracting
from the lips of the accused an admission of his
guilt.
It does not apply where the evidence sought to
be excluded is not an incriminating statement
but an OBJECT EVIDENCE (e.g. Fingerprinting,
photographing, paraffin testing, PE)
The prohibition extends to the compulsion for
the production of documents, papers and
chattels that may be used as evidence against
the witness except where the State has the right
to examine or inspect under the police power of
the State
The right also protects the accused against any
attempt to compel him to furnish a specimen of
his handwriting in connection with a prosecution
for falsification.

Das könnte Ihnen auch gefallen